Límite clásico de la integral de trayectoria de Feynman

Yo entiendo que en el limite que va a cero, la integral de trayectoria de Feynman está dominada por la trayectoria clásica, y luego usando la aproximación de fase estacionaria podemos derivar una aproximación para el propagador que es una función de la trayectoria clásica (ver http://www.blau.itp. unibe.ch/lecturesPI.pdf pág. 46).

Tengo la impresión de que esto implica además que la partícula sigue la trayectoria clásica, pero no entiendo cómo implica esto el hecho mencionado anteriormente.

El propagador describe la evolución temporal de la función de onda, por lo que creo que esta forma límite clásica del propagador debería dar una evolución temporal en la que la función de onda sigue la trayectoria clásica, pero no he podido encontrar tal trabajo. Además, incluso esta declaración en sí misma es problemática ya que la función de onda describe una distribución de probabilidad y no una única trayectoria.

Nueva edición: En la sección 7 del artículo de Feynman que presenta la integral de ruta (ver http://imotiro.org/repositorio/howto/artigoshistoricosordemcronologica/1948c%20-FEYNMAN%201948C%20Invention%20of%20the%20path%20integral%20formalism%20for%20quantum% 20mecánica.pdf ) analiza el límite clásico. Parece que la clave para comprender por qué el hecho de que la trayectoria clásica domine a la integral de trayectoria implica además que la partícula sigue la trayectoria clásica se puede encontrar en el comentario de Feynman en la página 21: "Ahora preguntamos, como 0 qué valores de las coordenadas intermedias X i contribuyen más fuertemente a la integral? Estos serán los valores más probables de encontrar por experimento y, por lo tanto, determinarán, en el límite, el camino clásico". Sin embargo, no entiendo por qué "Estos serán los valores más probables de encontrar por experimento".

Pregunta relacionada por OP: physics.stackexchange.com/q/32110/2451
Feynman simplemente quiere decir que las coordenadas X i los que contribuyan más fuertemente a la integral serán los que tengan mayor probabilidad si se pregunta por la posición de la partícula en algún momento intermedio.
Publicado de forma cruzada desde mathoverflow.net/q/102415

Respuestas (2)

El límite semiclásico que estás describiendo dice que la amplitud para que una partícula vaya de aquí para allá en un tiempo determinado es igual a la exponencial de la acción clásica para la trayectoria clásica correspondiente. En símbolos esto se lee

X b | tu ( T ) | X a = D φ mi i S [ ϕ ] / mi i S [ φ cl ( X a , X b , T ) ] / .
Sin embargo, en un estado cuántico general, las partículas no están "aquí" y no terminan "allá": tienen una amplitud de probabilidad inicial X | ψ ( 0 ) por estar en cada puesto X en el momento t = 0 y tendrá una amplitud de probabilidad final X | ψ ( T ) por estar en el puesto X en el momento T . Para aplicar la aproximación, extrae el propagador e inserta una resolución de la identidad:
X | ψ ( T ) = d y X | tu ( T ) | y y | ψ ( 0 ) = d y mi i S [ φ cl ( y , X , T ) ] / y | ψ ( 0 ) .

Para obtener un límite semiclásico completo, también necesita un estado inicial semiclásico (¡ya que de lo contrario obviamente no tiene esperanza!). Toma, entonces, un estado con una posición y un momento (relativamente) claramente definidos (por supuesto, el estado ocupará una región finita del espacio de fase, pero generalmente puede suponer, en estas circunstancias, que es lo suficientemente pequeño), y esto será hacer que las amplitudes de los puntos fuera de la trayectoria clásica interfieran destructivamente y desaparezcan.

EDITAR

Entonces, ¿cómo sucede esto? Para uno, y debe estar cerca de la posición inicial, y 0 para contribuir a la integral. Entonces, para pequeños desplazamientos de los puntos finales, la acción a lo largo de la trayectoria clásica varía como

d S = pag φ , X d X pag φ , y d y
(cf. Lanczos, The Variational Principles of Mechanics, 4th edition, Dover, eqs 53.3 and 68.1, o simplemente haga la integración estándar por partes y conjuntos d L d X = 0 a lo largo de la trayectoria clásica). La principal contribución del estado inicial a la fase es de la forma mi i pag cl y , lo que significa que la integral tiene más o menos la forma, salvo una fase,
X | ψ ( T ) y 0 Δ X / 2 y 0 + Δ X / 2 mi i ( pag φ , y pag cl ) y / d y .

Aquí el impulso pag φ , y Esta determinado por X y (al orden principal) y 0 , ya que existe un único camino clásico que los une. Este impulso debe coincidir (con precisión Δ pag / Δ X , que suponemos despreciable en este límite semiclásico) el momento clásico del estado inicial, pag cl , y por lo tanto sólo aquellos X 's en la trayectoria determinada por el estado inicial tendrán amplitudes distintas de cero.

Gracias por su respuesta. No entiendo por qué "esto hará que las amplitudes de los puntos fuera de la trayectoria clásica interfieran destructivamente y desaparezcan". Por favor Podría explicar esto. Además, vea mi nueva edición de la pregunta. Gracias.
Muchas gracias por su respuesta. Hay algunas cosas en las que podría usar más detalles para comprender mejor su respuesta. En lugar de entrar en preguntas detalladas, me preguntaba si conoce algún libro de texto o documento que analice su argumento. Gracias.
Realmente no conozco ninguna referencia con este material, pero me gustaría mucho ver una. También le estoy bastante agradecido por obligarme a resolver esto con claridad.

NO es cierto en general que el límite semiclásico siempre esté dominado por soluciones estacionarias del Lagrangiano desnudo original. En cambio, el mejor límite cuasiclásico se parece más al marco de historias consistentes de grano grueso, también conocido como historias decoherentes a lo largo de la base de puntero determinada dinámicamente.